What is the solution to the system of linear equations?
(-3,0)
(-3,3)
(0,2)
(3,1)

What Is The Solution To The System Of Linear Equations?(-3,0)(-3,3)(0,2)(3,1)

Answers

Answer 1
(0,2) that’s the point in which they connect or meet

Related Questions

Instructions: Solve the following linear
equation.
- 2x + 38 = 2(3 + 3x)
2-

Answers

Answer:

Step-by-step explanation:

-2x +38 = 2(3 + 3x)

-2x + 38 = 2*3 + 2*3x

-2x + 38 = 6 + 6x

Add 2x to both sides

38 = 6 + 6x + 2x  

Combine like terms

38 = 6 + 8x

Subtract 8  from both sides

38 - 6 = 8x

8x = 32

Divide both sides by 8

x = 32/8

x = 4

Answer:

x = 4

Step-by-step explanation:

-2x + 38 = 2(3 + 3x)

Use distributive property to multiply 2 by 3 and 3x

-2x + 38 = 2 ×3 + 2 × 3x -2x + 38 = 6 + 6x

subtract 6x from both side

-2x + 38 - 6x = 6 + 6x - 6x

combine -2x and -6x to get -8x

-8x + 38 = 6

subtract 38 from both side

-8x + 38 - 38 = 6 - 38

subtract 38 from 6 to get -32

-8x = -32

divide both side by -8

[tex] \small \sf \frac{-8x}{ -8} = \frac{-32}{-8} \\ \\ \small \sf x = \frac{- 32}{-8}[/tex]

divide -32 by -8 to get 4

x = 4

You are certain to get a heart, diamond, club, or spade when selecting cards from a shuffled deck. Express the indicated degree of likelihood as a probability value between 0 and 1 inclusive.

Answers

Answer:

The probability of this event is represented by a value of 1.

Step-by-step explanation:

Probability of a certain event:

The probability of an event that is considered to be certain, that is, guaranteed to happen, is 100% = 1.

You are certain to get a heart, diamond, club, or spade when selecting cards from a shuffled deck.

This means that the probability of this event is represented by a value of 1.

Explain the difference between a rate, a ratio, and a proportion?

Answers

Answer:

A proportion is an equality of two ratios.

Example : [tex]\frac{1}{3}[/tex] = [tex]\frac{x}{9}[/tex]

We write proportions to help us find equivalent ratios and solve for unknown quantities.

A rate is the quotient of a ratio where the quantities have different units.

Example : [tex]\frac{distance}{time}[/tex]

A ratio is a comparison of two quantities.

Example : 1 : 3 or [tex]\frac{1}{3}[/tex]

Write an expression for the sequence of operations described below.
divide s by u, add the result to t, then add v to what you have
Do not simplify any part of the expression

Answers

Step-by-step explanation:

s+u

Add t: You will need to introduce brackets

(s+u) + t

Add v: Introduce another v

((s/u) + t) + v

consider this equation sin(theta) = -4 square root of 29/29 if theta is an angle in quadrant IV what is the value of cos (theta)

Answers

Answer:

[tex]\cos(\theta)= \frac{\sqrt{377}}{29}[/tex]

Step-by-step explanation:

Given

[tex]\sin(\theta) = -\frac{4}{\sqrt{29}}[/tex] -- the correct expression

Required

[tex]\cos(\theta)[/tex]

We know that:

[tex]\sin^2(\theta) + \cos^2(\theta)= 1[/tex]

Make [tex]\cos^2(\theta)[/tex] the subject

[tex]\cos^2(\theta)= 1 - \sin^2(\theta)[/tex]

Substitute: [tex]\sin(\theta) = -\frac{4}{\sqrt{29}}[/tex]

[tex]\cos^2(\theta)= 1 - (-\frac{4}{\sqrt{29}})^2[/tex]

Evaluate all squares

[tex]\cos^2(\theta)= 1 - (\frac{16}{29})[/tex]

Take LCM

[tex]\cos^2(\theta)= \frac{29 - 16}{29}[/tex]

[tex]\cos^2(\theta)= \frac{13}{29}[/tex]

Take square roots of both sides

[tex]\cos(\theta)= \±\sqrt{\frac{13}{29}}[/tex]

cosine is positive in the 4th quadrant;

So:

[tex]\cos(\theta)= \sqrt{\frac{13}{29}}[/tex]

Split

[tex]\cos(\theta)= \frac{\sqrt{13}}{\sqrt{29}}[/tex]

Rationalize

[tex]\cos(\theta)= \frac{\sqrt{13}}{\sqrt{29}} * \frac{\sqrt{29}}{\sqrt{29}}[/tex]

[tex]\cos(\theta)= \frac{\sqrt{13*29}}{29}[/tex]

[tex]\cos(\theta)= \frac{\sqrt{377}}{29}[/tex]

Given that Z1 = 1 + i and Z2 = 3 - 4i, find z1z2

Answers

Answer:

7-i

Step-by-step explanation:

It is asking for the product of the given complex numbers.

Z1Z2 means Z1 times Z2

(1+i)(3-4i)

You can do the whole foil thing here since we are multiplying a pair of binomials. But all you are doing when you do that is multiplying every term in the first ( ) to every term in the second ( ).

1(3)+1(-4i)+i(3)+i(-4i)

Simplify each term. That is, perform the multiplication in each term:

3-4i+3i-4i^2

Combine like terms and also replace i^2 with (-1):

3-1i-4(-1)

Multiplication identity property used:

3-i+4

Combine like terms:

7-i

The following is a scatterplot of the percent of children under age 18 who are not in school or in the labor force vs. the number of juvenile violent crime arrests for each of the 50 states. The least-squares regression line has been drawn in on the plot. We would like to predict what the number of juvenile violent crime arrests would be in a state if 25% of children are not in school or in the labor force. This is called

Answers

Answer:

Extrapolation

Step-by-step explanation:

From the linear regression plot created in the picture given, se could see that Tha percentage of student covered by the the plot is just above 16%. Therefore, to predict the percentage of the number of juvenile violent crime arrests would be in a state if 25% of children are not in school or in the labor force will require us to assume that the current trend continues into the future. Hence, we use the information and indications we have at present to make prediction into the future based on the assumption that we the current trend will remain relevant and applicable. This assumption into the future based on current trend is called EXTRAPOLATION.

What is the midpoint of the segment shown below?

Answers

Answer:

A

Step-by-step explanation:

Midpoints are found by averaging the coordinates.

Averaging " add all the numbers and divide by the number of numbers.

Here, there are only 2 numbers. So, you divide by 2.

(1,2) (1,-5)

[tex]\frac{1 +1}{2}[/tex] , [tex]\frac{2 + (-5)}{2}[/tex]

[tex]\frac{2}{2}[/tex] , [tex]\frac{-3}{2}[/tex]

(1, [tex]\frac{-3}{2}[/tex] )

The owners of a baseball team are building a new baseball field for their team and must determine the number of seats to include. The average game is attended by 6,500 fans, with a standard deviation of 450 people. Suppose a random sample of 35 games is selected to help the owners decide the number of seats to include. Identify each of the following and be sure to round to the nearest whole number:
Provide your answer below:
μ =------------
μx=-----------
σx=-----------
σ=------------
n=------------

Answers

Answer:

μ = 6500

μx= 6500

σx= 76

σ= 450

n= 35

Step-by-step explanation:

Central Limit Theorem

The Central Limit Theorem establishes that, for a normally distributed random variable X, with mean [tex]\mu[/tex] and standard deviation [tex]\sigma[/tex], the sampling distribution of the sample means with size n can be approximated to a normal distribution with mean [tex]\mu[/tex] and standard deviation [tex]s = \frac{\sigma}{\sqrt{n}}[/tex].

For a skewed variable, the Central Limit Theorem can also be applied, as long as n is at least 30.

The average game is attended by 6,500 fans, with a standard deviation of 450 people.

This means that [tex]\mu = 6500, \sigma = 450[/tex]

35 games:

This means that [tex]n = 35[/tex]

Distribution of the sample mean:

By the Central Limit Theorem, we have [tex]\mu_x = \mu = 6500[/tex] and the standard deviation is:

[tex]\sigma_x = \frac{450}{\sqrt{35}} = 76[/tex]

Solve the system of linear equations below.
6x + 3y = 33
4x + y = 15

A.
x = 2, y = 7
B.
x = -13, y = 7
C.
x = - 2/3, y = 12 2/3
D.
x = 5, y = 1

Answers

Answer:

The answer for both linear equations is A. x = 2, y = 7

Step-by-step explanation:

First start by plugging in the variables with the given numbers (2,7). We'll start with 6x + 3y = 33.

6x + 3y = 33

6 (2) + 3 (7 )= 33 <--- This is the equation after the numbers are plugged in.

12 + 10 = 33

33 = 33 <---- This statement is true, therefore it is the correct pair.

Now we are not done, to confirm that this pair works with both equations we need to solve for 4x + y = 15 to see if it works. Linear Equations must have the variables work on both equations.

4x + y = 15 <----- We are going to do the exact same thing to this equation.

4(2) + 7 = 15

8 + 7 = 15

15 = 15  <-- 15=15 is a true statement therefore this pair works for this equation.

Therefore,

A. x = 2, y = 7 is the correct answer

Sorry this is a day late, I hope it helps.

How much do I need to subtract from 67/10 to make 6

Answers

Answer:

0.7

Step-by-step explanation:

67/10 is the same as 6.7 when you subtract the 0.7 you will remain with 6

Express the following composite numbers as products of prime factors 64

Answers

Answer:

64 is a composite number, and it is 8 squared. 64 = 1 x 64, 2 x 32, 4 x 16, or 8 x 8. Factors of 64: 1, 2, 4, 8, 16, 32, 64. Prime factorization: 64 = 2 x 2 x 2 x 2 x 2 x 2, which can also be written 64 = 2⁶.

Pierre Martina is comparing the cost of credit to the cash price of an item. If Pierre makes a down payment of $70 and pays $34 a
month for 24 months, how much more will that amount be than the cash price of $695? (Input the amount as a positive value.)
Amount of difference

Answers

Answer:

$191

Step-by-step explanation:

Find how much she will be paying over the 24 months:

34(24)

= 816

Add the down payment to this:

816 + 70

= 886

Find the difference between this and 695:

886 - 695

= 191

So, the difference will be $191 more

Exponential and Alogarithmic Functions - Alegebra question

Answers

Answer:

Step-by-step explanation:

Given the number of hours spent amusing ourselves to death with screen time, how many Titanics could we have built in one year if all those hours had been spent building Titanics

Answers

Answer:

12

Step-by-step explanation:

5. Determine the total area of the figure below.
4 ft!
12 ft
10 ft
3 ft
9 ft

Answers

Area: 171ft^2

Explanation:
The triangles area is 24ft^2
The large rectangle is 120ft^2
The small rectangle is 27ft^2
24+120+27= 171ft^2

Please help !! Only answer if 100% it is correct :)

Answers

Answer:

F(x) moved right 2 units to become G(X).

According to graph transformations, that means G(X) = F(X - 2) = [tex](X-2)^{3}[/tex].

I think that's how you do it :\

NO LINKS OR ANSWERING WHAT YOU DON'T KNOW?

1. Suppose y varies inversely with x, and y = 25 when x = 1/5. What is the value of y when x = 5?

a. 15
b. 5
c. 25
d. 1

2. Suppose y varies inversely with x, and y = a when x = a^2. What inverse variation equation related x and y?

a. y = a^2/x
b. y = a^3/x
c. y= a^3x
d. y = ax

3. Suppose y varies inversely with x, and y = 3 when x = 1/3. What is the inverse variation equation that relates x and y?

a. y = 1/x
b. y =x
c. y = 3x
d. y = 3/x

Answers

Answer:

1. D. 1

2. B. y=a³/x

3. A. y=1/x

Step-by-step explanation:

too long to give te explanations but they're there in the attachments

Find the distance between the two points in simplest radical form (−6, 1) and (−8,−4)

Answers

=√(-8-(-6))^+(-4-1)^=√(-2)^+(-5)^= -2-5= -7

Johnny tripled his baseball card collection. Then he added 6 more cards to the collection. Now he has 24 cards. How many cards did he start with?

Answers

9514 1404 393

Answer:

  6

Step-by-step explanation:

Work backward.

If he has 24 after adding 6, he had 18 before that addition.

If he had 18 after tripling his collection, he had 18/3 = 6 cards to start with.

__

Note that this is the same process you would use if you started with an equation.

  3c +6 = 24 . . . . where c is the number of cards Johnny started with

  3c = 24 -6 = 18 . . . . . subtract 6 from the final number

  c = 18/3 = 6 . . . . . . . . divide the tripled value by 3 to see the original value

Johnny started with 6 cards.

13. Given that
[tex] {x}^{2} + {y}^{2} + 10y + 16 = 0[/tex]
and
[tex] {(x - 3)}^{2} + {y}^{2} = 1[/tex] are two circles on the same plane. Find:
a) the coordinates of the center and the radius for each circle.
b) the equation of the straight line joining the center of both circles.​

Answers

step by step explanation:

[tex]\mathfrak{x}^{2}+{y}^{2}+16=0[/tex]

=[x2+16=0x26]

=[2x{y}^2{16}~0]

=[4×{y}^0{16}]

=[32x{y}^x]

A mechanic charges $65 for an engine check and $20 per hour for his
services. Which of the following is a linear model of his charges.
y=20x+65
y=65x+20
y=3.25x+65
O y=3.25x+20
Question 5

Answers

Answer:

y = 65 + 20x

Step-by-step explanation:

Okay, when you're talking about linear equations try to find the fixed value, and then the changing one.

The fixed value will be by itself

The value that varies will have a variable next to it (x, y, z, whatever)

Then, the answer has to be

y = 65 + 20x

If the range of the coordinate transformation (, ) = (−2,−3 +1) is (4, −2), (2, −5), (−6, 4), what is the domain?

A. (-2, 1), (-1, 2), (3, -1)

B. (-8, 7), (-4, 16), (19, -11)

C. (-8, 1), (-4, 2), (19, -1)

D. (-2, 7), (-1, 16), (3, -11)

Answers

Consider the below figure attached with this question.

Given:

The transformation is:

[tex]f(x,y)=(-2x,-3y+1)[/tex]

The range is (4,-2), (2, −5), (−6, 4).

To find:

The domain of the transformation.

Solution:

We have,

[tex]f(x,y)=(-2x,-3y+1)[/tex]

For the point (4,-2),

[tex](-2x,-3y+1)=(4,-2)[/tex]

On comparing both sides, we get

[tex]-2x=4[/tex]

[tex]x=\dfrac{4}{-2}[/tex]

[tex]x=-2[/tex]

And,

[tex]-3y+1=-2[/tex]

[tex]-3y=-2-1[/tex]

[tex]-3y=-3[/tex]

[tex]y=\dfrac{-3}{-3}[/tex]

[tex]y=1[/tex]

So, the domain of (4,-2) is (-2,1).

Similarly,

For the point (2,-5),

[tex](-2x,-3y+1)=(2,-5)[/tex]

On comparing both sides, we get [tex]x=-1,y=2[/tex]. So, the domain of (2,-5) is (-1,2).

For the point (-6,4),

[tex](-2x,-3y+1)=(-6,4)[/tex]

On comparing both sides, we get [tex]x=3,y=-1[/tex]. So, the domain of (-6,4) is (3,-1).

So, the domain of the given transformation is (-2, 1), (-1, 2), (3, -1).

Therefore, the correct option is A.

How do you complete the other two?
I know how to complete the first one but 3D Pythag confuses me so much

Answers

For now, I'll focus on the figure in the bottom left.

Mark the point E at the base of the dashed line. So point E is on segment AB.

If you apply the pythagorean theorem for triangle ABC, you'll find that the hypotenuse is

a^2+b^2 = c^2

c = sqrt(a^2+b^2)

c = sqrt((8.4)^2+(8.4)^2)

c = 11.879393923934

which is approximate. Squaring both sides gets us to

c^2 = 141.12

So we know that AB = 11.879393923934 approximately which leads to (AB)^2 = 141.12

------------------------------------

Now focus on triangle CEB. This is a right triangle with legs CE and EB, and hypotenuse CB.

EB is half that of AB, so EB is roughly AB/2 = (11.879393923934)/2 = 5.939696961967 units long. This squares to 35.28

In short, (EB)^2 = 35.28 exactly. Also, (CB)^2 = (8.4)^2 = 70.56

Applying another round of pythagorean theorem gets us

a^2+b^2 = c^2

b = sqrt(c^2 - a^2)

CE = sqrt( (CB)^2 - (EB)^2 )

CE = sqrt( 70.56 - 35.28 )

CE = 5.939696961967

It turns out that CE and EB are the same length, ie triangle CEB is isosceles. This is because triangle ABC isosceles as well.

Notice how CB = CE*sqrt(2) and how CB = EB*sqrt(2)

------------------------------------

Now let's focus on triangle CED

We just found that CE = 5.939696961967 is one of the legs. We know that CD = 8.4 based on what the diagram says.

We'll use the pythagorean theorem once more

c = sqrt(a^2 + b^2)

ED = sqrt( (CE)^2 + (CD)^2 )

ED = sqrt( 35.28 + 70.56 )

ED = 10.2878569196893

This rounds to 10.3 when rounding to one decimal place (aka nearest tenth).

Answer: 10.3

==============================================================

Now I'm moving onto the figure in the bottom right corner.

Draw a segment connecting B to D. Focus on triangle BCD.

We have the two legs BC = 3.7 and CD = 3.7, and we need to find the length of the hypotenuse BD.

Like before, we'll turn to the pythagorean theorem.

a^2 + b^2 = c^2

c = sqrt( a^2 + b^2 )

BD = sqrt( (BC)^2 + (CD)^2 )

BD = sqrt( (3.7)^2 + (3.7)^2 )

BD = 5.23259018078046

Which is approximate. If we squared both sides, then we would get (BD)^2 = 27.38 which will be useful in the next round of pythagorean theorem as discussed below. This time however, we'll focus on triangle BDE

a^2 + b^2 = c^2

b = sqrt( c^2 - a^2 )

ED = sqrt( (EB)^2 - (BD)^2 )

x = sqrt( (5.9)^2 - (5.23259018078046)^2 )

x = sqrt( 34.81 - 27.38 )

x = sqrt( 7.43 )

x = 2.7258026340878

x = 2.7

--------------------------

As an alternative, we could use the 3D version of the pythagorean theorem (similar to what you did in the first problem in the upper left corner)

The 3D version of the pythagorean theorem is

a^2 + b^2 + c^2 = d^2

where a,b,c are the sides of the 3D block and d is the length of the diagonal. In this case, a = 3.7, b = 3.7, c = x, d = 5.9

So we get the following

a^2 + b^2 + c^2 = d^2

c = sqrt( d^2 - a^2 - b^2 )

x = sqrt( (5.9)^2 - (3.7)^2 - (3.7)^2 )

x = 2.7258026340878

x = 2.7

Either way, we get the same result as before. While this method is shorter, I think it's not as convincing to see why it works compared to breaking it down as done in the previous section.

Answer:  2.7

Answer:

Qu 2    =  10.3 cm

Qu 3.   = 2.7cm

Step-by-step explanation:

Qu 2. Shape corner of a cube

We naturally look at sides for slant, but with corner f cubes we also need the base of x and same answer is found as it is the same multiple of 8.4^2+8/4^2 for hypotenuse.

8.4 ^2 + 8.4^2 = sq rt 141.42 = 11.8920141 = 11.9cm

BD = AB =  11.9 cm  Base of cube.

To find height x we split into right angles

formula slant (base/2 )^2 x slope^2  = 11.8920141^2 - 5.94600705^2 =  sq rt 106.065

= 10.2987863

height therefore is x = 10.3 cm

EB = 5.9cm

BC = 3.7cm

CE^2  = 5.9^2 - 3.7^2  = sqrt 21.12 = 4.59565012 = 4.6cm

2nd triangle ED = EC- CD

= 4.59565012^2- 3.7^2 = sq rt 7.43000003 =2.72580264

ED = 2.7cm

x = 2.7cm

5. There are 8 sections of seats in an auditorium. Each section contains at least 150 seats but not more than 200 seats. Which of the following could be the number of seats in this auditorium? (A) 800 (B) 1,000 (C) 1,100 (D) 1,300 (E) 1,700​

Answers

Answer:

1300

Step-by-step explanation:

Given that :

Number of sections = 8

Number of seats per section, x ;

150 ≤ x ≤ 200

The possible Number of seats in the auditorium :

Number of seats per section * number of sections

150 * 8 ≤ x ≤ 200 * 8

1200 ≤ x ≤ 1600

The possible Number of seats will lie within 1200 and 1600

From the options, only 1300 lie within this range

find the rate of change of volume of a cone if dr/dt is 3 in./min. and h=4r when r = 8 inches

Answers

Answer:

[tex]v = \frac{1}{3}bh[/tex]

since base is pi r^2

[tex]v = \frac{1}{3} \pi \: r {}^{2} h[/tex]

it's given that h=4r

[tex]v = \frac{1}{3} \pi \: r^{2} (4r) = \frac{4}{3} \pi \: {r}^{3} [/tex]

now find derivative

[tex] \frac{dv}{dt} = 4\pi \: r {}^{2} [/tex] × dr/dt

r=8 , dv/dt = 3

dv/dt = 4pi (8)^2 ×3 = 768pi

Answer:

768 pi in^3/min

Step-by-step explanation:

Volume of cone=1/3 pi×r^2×h

Differentiating this gives:

dV/dt=1/3×pi×2r dr/dt×h+1/3×pi×r^2×dh/dt

We are given the following:

dr/dt = 3 in./min.

h=4r when r = 8 inches

If h=4r then dh/dt=4dr/dt=4(3 in/min)=12 in/min

If h=4r and r=8 in, then h=4(8)=32 in for that particular time.

Plug in:

dV/dt=1/3×pi×2r dr/dt×h+1/3×pi×r^2×dh/dt

dV/dt=1/3×pi×2(8)(3)×32+1/3×pi×(8)^2×12

dV/dt=pi×2(8)(32)+pi×(8)^2(4)

dV/dt=pi(256×2)+pi(64×4)

dV/dt=pi(512)+pi(256)

dV/dt=pi(768)

dV/dt=768pi

dV/dt=768/pi in^3/min

express ratio as a fraction in it's lowest terms.48s to 5minutes​

Answers

Answer:

4/25.

Step-by-step explanation:

We need to have the 2 values in the same unit so:

converting minutes to seconds:

5 mins = 5*60 = 300 seconds.

The require fraction =

48/300              

The GCF of 48 and 300 is 12 so we have:

48/12 / 300/12  

= 4/25

The required fraction of 48 seconds to 5 minutes is  4 / 5 minutes.

Given that,
To express the ratio as a fraction in its lowest terms.48s to 5 minutes​.

What is arithmetic?

In mathematics, it deals with numbers of operations according to the statements. There are four major arithmetic operators, addition, subtraction, multiplication, and division,

Here,
60 second = 1 minute
1 second = 1 / 60 minute
multiply both sides by 48
48 second = 48 / 60 minute
48 second = 4 / 5 minute


Thus, the required fraction of 48 seconds to 5 minutes is  4 / 5 minutes.

Learn more about arithmetic here:

brainly.com/question/14753192

#SPJ2

help please i don’t understand it at this moment

Answers

Answer:

it's H. 1/2 in.=1,000 ft

F. 1 in.= 100ft

[tex]{hope 8 helps}}[/tex]

Movie genres. The pie chart summarizes the genres of 120 first-run movies released in 2005. a) Is this an appropriate display for the genres

Answers

Answer:

Yes, it is appropriate

Step-by-step explanation:

Given

See attachment for pie chart

Required

Is the pie chart appropriate

The attached pie chart displays the distribution of each of the 4 genre. The partition occupied represents the measure of each genre.

The half-life of a newly discovered radioactive element is 30 seconds. To the nearest tenth of a second, how long will it take for a sample of 9 grams to decay to 0.72 grams

Answers

Answer:

It will take about 109.3 seconds for nine grams of the element to decay to 0.72 grams.

Step-by-step explanation:

We can write a half-life function to model our function.

A half-life function has the form:

[tex]\displaystyle A=A_0\left(\frac{1}{2}\right)^{t/d}[/tex]

Where A₀ is the initial amount, t is the time that has passes (in this case seconds), d is the half-life, and A is the amount after t seconds.

Since the half-life of the element is 30 seconds, d = 30. Our initial sample has nine grams, so A₀ is 9. Substitute:

[tex]\displaystyle A=9\left(\frac{1}{2}\right)^{t/30}[/tex]

We want to find the time it will take for the element to decay to 0.72 grams. So, we can let A = 0.72 and solve for t:

[tex]\displaystyle 0.72=9\left(\frac{1}{2}\right)^{t/30}[/tex]

Divide both sides by 9:

[tex]\displaystyle 0.08=\left(\frac{1}{2}\right)^{t/30}[/tex]

We can take the natural log of both sides:

[tex]\displaystyle \ln(0.08)=\ln\left(\left(\frac{1}{2}\right)^{t/30}\right)[/tex]

By logarithm properties:

[tex]\displaystyle \ln(0.08)=\frac{t}{30}\ln(0.5)[/tex]

Solve for t:

[tex]\displaystyle t=\frac{30\ln(0.08)}{\ln(0.5)}\approx109.3\text{ seconds}[/tex]

So, it will take about 109.3 seconds for nine grams of the element to decay to 0.72 grams.

Other Questions
An oxygen atom with three missing electrons is released near the Van de Graaff generator. What is its energy in MeV at this distance write an equation in slope intercept form for the line with slope 1/4 and y-intercept -6. The Students in a school can be arranged in 12, 15, 18 equal rows and also into a solid square. What is the lowest number of students that can be in the school? (Hint: find the LCM) A person you recently met in one of your classes tells you that she was adopted shortly after her birth. This is an example of Which statements are consistent with Dalton"s atomic theory as it was originally stated? Why? a. Sulfur and oxygen atoms have the same mass. b. All cobalt atoms are identical. c. Potassium and chlorine atoms combine in a 1:1 ratio to form potass... Marcella incarica Dora di aiutarla a suddividere 35 perle tra le sue tre figlie. Alla figlia maggiore vuole regalare la met, alla mediana la terza parte e alla minore la nona parte. Inoltre Marcella vuole dare una perla a Dora come ricompensa. Quante perle ricever ciascuna figlia? New technologies such as ________ and ________ led to a new boom era of oil and gas production in Texas beginning in 2008 and continuing through today. Select the correct answer.Tom gets $12 off a box of chocolates that had an original price of $48. What percentage is the discount Senario: 2 years ago, a fruit was smuggled into California on a plane from an exotic, far away land. The homeowner saw that the fruit had maggots and tossed it into the backyard, hoping the seed would grow. The larvae hatched out and moved throughout the area. This fictitious insect will destroy fruit and has the possibility of spreading disease killing the trees. The insect consumes plants in the Prunus species of stone fruits? Look up the plant genus Prunus. Discussion: The insect has spread over a large area of Southern California, discovered at UC Riverside. What steps would you do to control or eradicate the destructive exotic insect?PLZ HELP THX WITH COLLEGE LEVEL EXPERICENCE what is the range of this function 6 goes to 5 8 goes to -3 10goes to -8 and 12 goes to 7a.(-3,7,8,12)b.(-8,-3,5,6,7,8,10,12)c.(-8,-3,5,7)d.(6,8,10,12) write a function rule for the following data Calculate the solubility (in mol/L) of Fe(OH)3 (Ksp = 4.0 x 10^-38) in each of the following situations: (A) Pure water (assume that the pH is 7.0 and remains constant). (B) A solution buffered at pH = 5.0. (C) A solution buffered at pH = 11.0. What are 5 characteristics of hobbits? find the slope of the tangent line [tex]m_{tan}[/tex] = f'(a) and then find the equation of the tangent line to f at x = af(x) = [tex]\frac{10}{x}[/tex] ; a = 3 Which behaviors show healthy boundaries in a romantic relationship?Select all that apply.O expecting others to take responsibility for their own actionsO making time for other friends and independent interestsexpecting respect from a romantic partnershetrying to change a romantic partner to fit your desires Help please I dont get may I please have help with this Use the initial term and the recursive formula to find an explicit formula for the sequence an. Write your answer in simplest form. What does the verse in Song of Solomon 5:16 mean His mouth is most sweet thank you very much What is the expression (x+1)(x2+3x+2) as a polynomial in standard form? i need help please and thanks